PiR2
Gostaria de reagir a esta mensagem? Crie uma conta em poucos cliques ou inicie sessão para continuar.

Questão Hidrostática

2 participantes

Ir para baixo

Questão Hidrostática Empty Questão Hidrostática

Mensagem por Joca0004 Sáb 06 Ago 2022, 14:05

São dados: Um tubo cilindrícco, de raio constante, R, recurvado em forma de U; um êmbolo E, que pode se deslocar sem atrito ao longo da parte horizontal do ttubo; dois líquidos conhecidos 1 e 2, colocados a esquerda e a direita do êmbolo. Sabendo-se que o êmbolo fica em equilíbrio quando; H=2h. Calcular de quanto ele se deslocará quando o volume do liquido 2 for aumentado do valor [latex]\Delta V= pi R^3[/latex]


(A) R
(B) R/2
(C) R/3
(D) R/4
(E) R/5

Joca0004
Iniciante

Mensagens : 3
Data de inscrição : 21/02/2022

Ir para o topo Ir para baixo

Questão Hidrostática Empty Re: Questão Hidrostática

Mensagem por Leonardo Mariano Sáb 06 Ago 2022, 17:01

Com a primeira informação sobre o equilíbrio, podemos encontrar uma relação entre as densidades dos líquidos 1 e 2.
Entretanto, não foi informado a qual líquido se refere H = 2h, então vamos fazer as duas possibilidades e ver o que aparece.

a) Liquido 1 = 2h e líquido 2 = h:

[latex] p_1gH=p_2gh\rightarrow p_1g2h=p_2gh\rightarrow 2p_1=p_2 \:(I) [/latex]

Ao adicionar o volume informado ao líquido 2, ele inicialmente subirá uma altura h':

[latex] V_{tubo}=V_{adicionado}\rightarrow \pi R^2h'=\pi R^3\rightarrow h' = R [/latex]

Ou seja, o volume informado equivale a um acréscimo de uma coluna R de líquido.
Para voltar ao equilíbrio, é necessário que o líquido 2 desça, e como o tubo tem o mesmo raio em ambos os lados, o líquido 1 subirá a mesma distância:

[latex] P_1 = P_2\rightarrow p_1g(2h + x)=p_2g(h + R - x) 
\rightarrow 2p_1gh + p_1gx=2p_1gh + 2p_1gR - 2p_1gx 
\rightarrow 3p_1gx=2p_1gR\rightarrow x=\frac{2R}{3} [/latex]

Ou seja, o líquido 2 desceu uma altura 2R/3. O movimento do êmbolo segue o dos líquidos, então esse é o seu deslocamento.
Entretanto, não há essa resposta. Indo para o caso 2:

b) Liquido 1 = h e líquido 2 = 2h:
Apenas repetindo os cálculos você encontra isso:

[latex] 2p_2=p_1 \:(I) [/latex]

Deslocamento:

[latex] P_1 = P_2\rightarrow p_1g(h + x)=p_2g(2h + R - x) \therefore x = \frac{R}{3} [/latex]

Creio que seja isso, você tem o gabarito?
Leonardo Mariano
Leonardo Mariano
Monitor
Monitor

Mensagens : 493
Data de inscrição : 11/11/2018
Idade : 22
Localização : Criciúma/SC

Joca0004 gosta desta mensagem

Ir para o topo Ir para baixo

Questão Hidrostática Empty Re: Questão Hidrostática

Mensagem por Joca0004 Dom 07 Ago 2022, 16:08

É isso mesmo, show. O líquido 2 = 2h e o 1 = h. Obg

Joca0004
Iniciante

Mensagens : 3
Data de inscrição : 21/02/2022

Leonardo Mariano gosta desta mensagem

Ir para o topo Ir para baixo

Questão Hidrostática Empty Re: Questão Hidrostática

Mensagem por Conteúdo patrocinado


Conteúdo patrocinado


Ir para o topo Ir para baixo

Ir para o topo

- Tópicos semelhantes

 
Permissões neste sub-fórum
Não podes responder a tópicos